Relativity paradox

Discussion in 'Physics & Math' started by renislaj, Apr 18, 2013.

  1. Markus Hanke Registered Senior Member

    Messages:
    381
    Thank you, but I must qualify that post by saying that this is my own solution to the puzzle. That is not to say that there are not other, better ones. The point is simply that the physical outcome as seen from the two frames cannot differ.

    Yes, I can see that. Since he hasn't learned at thing in 2700+ posts, I think it is safe to say that responding to him is a complete waste of time.
     
  2. Google AdSense Guest Advertisement



    to hide all adverts.
  3. Markus Hanke Registered Senior Member

    Messages:
    381
    I agree.
     
  4. Google AdSense Guest Advertisement



    to hide all adverts.
  5. Motor Daddy Valued Senior Member

    Messages:
    5,425
    Too much avoidance of my questions to have any credibility with me. When you answer my direct questions then we can talk. Until then I am wasting my time with you. Carry on.
     
  6. Google AdSense Guest Advertisement



    to hide all adverts.
  7. eram Sciengineer Valued Senior Member

    Messages:
    1,877
    Motor Daddy denies just about every single thing.

    Please Register or Log in to view the hidden image!

     
    Last edited: Apr 18, 2013
  8. eram Sciengineer Valued Senior Member

    Messages:
    1,877
    At the end, they all do, with meat hooks. He also finds out the police are covering it up.
     
  9. Markus Hanke Registered Senior Member

    Messages:
    381
    Why would it matter if we have credibility with your or not ??
     
  10. Motor Daddy Valued Senior Member

    Messages:
    5,425

    For instance, one rainy night I am driving down a dark road and I see somebody, standing on the side of the road, soaking wet, cold, and miles from home. I stop, put down my driver's window and notice it's you. I quickly remember that you couldn't care less about me, so I adjust my actions accordingly by putting up the window and driving off, leaving you to your misery.
     
  11. Markus Hanke Registered Senior Member

    Messages:
    381
    That's just fine, because what you will find is that the two frames of reference are perfectly symmetric - swap the guy on the roadside with yourself in the car, and the physical outcome of the scenario would be the exact same.

    Relativity works !!!!

    Please Register or Log in to view the hidden image!

     
  12. Motor Daddy Valued Senior Member

    Messages:
    5,425

    See, you are so far up Einstein's butt that you can't see the light! You do not own my car and you are not the one driving, I am! You are the one on the side of the road that's cold and wet!
     
  13. eram Sciengineer Valued Senior Member

    Messages:
    1,877
    Ah, a most practical application of relativity.

    In the freezing rain, or in a cramped car with Motor Daddy? That's a tough decision.

    Please Register or Log in to view the hidden image!






    An artichoke is probably easier to reason with.

    Please Register or Log in to view the hidden image!



    Please Register or Log in to view the hidden image!

     
  14. RJBeery Natural Philosopher Valued Senior Member

    Messages:
    4,222
    Markus Hanke's explanation is interesting but I believe the correct answer is much simpler. The body's end points hitting the ground are spatially-separated events; there is only a single frame in theory which would claim that they happen simultaneously. The OP makes the presumption that all frames should agree on the concept of simultaneity...the impossibility of which is one of the very first elementary lessons to be deduced from SR.
     
  15. Neddy Bate Valued Senior Member

    Messages:
    2,548
    I'm not sure what you mean by "Falls down straight", so I will just have to give you the benefit of the doubt on that one. However, one thing is for sure, according to the embankment frame, the head of the body does not hit the floor at the same time as the feet. That is the whole significance of relativity of simultaneity. There is no paradox though, because the embankment frame can easily transform the time coordinates of the impacts of the head and feet and determine they are simultaneous in the train frame. Thus, the head does not absorb the full impact of the fall. That is the beauty of the Lorentz transformations.
     
  16. Tach Banned Banned

    Messages:
    5,265
    Yes, so what? This is given from the beginning.


    This doesn't follow. In the embankment frame one end would hit first, so you cannot "transform away" that effect, the same way one cannot "transform away" gravitational fields in GR.

    The Lorentz transforms are very beautiful indeed but you haven't proven anything of any value in this case.
     
  17. Tach Banned Banned

    Messages:
    5,265
    No, it doesn't. There is no implication in the OP about agreeing on simultaneity, there is a different implication, though. Could you figure what that is?
     
  18. Neddy Bate Valued Senior Member

    Messages:
    2,548
    How would the embankment frame explain the head not absorbing the entire impact? I propose that they would transform the time coordinates to the train frame, and determine that the impact of the feet happened at the same time, thus absorbing some of the impact. If you have a better explanation, why not post it?
     
  19. Tach Banned Banned

    Messages:
    5,265
    By the fact that , in the embankment frame, if one considers the effect of RoS alone, one end of the body hits before the other end. It is pretty clear in the OP, read it.



    Nope, you need to take each frame at face value, you keep trying to transform away the effect in the embankment frame. I already explained that you are not allowed to do that.


    Re-read Markus' explanation at post 2. I agree with him.
     
  20. RJBeery Natural Philosopher Valued Senior Member

    Messages:
    4,222
    I was hasty in my post. I thought the objection in the OP was that the body did not fall "straight down" in both frames. It appears that the objection is that the impact on each end of the body would apparently differ in one frame; let's replace the body with a steel I-beam and on the floor let's put a series of accelerometers; do the accelerometers' readings equal in all frames after the entire I-beam has impacted the floor? Of course, they must. I did not appreciate Markus' explanation because he appeared to be saying that Thomas precession would guarantee that the body would hit the ground in parallel fashion; this part cannot be true. At relativistic speeds the body [I-beam] does not remain straight when there is torque involved. However, perhaps Thomas precession could introduce a negative torque in such a way as to perfectly balance the impact of each end of the body [I-beam] on each accelerometer on the floor...even if those readings did not occur simultaneously in all frames.
     
  21. Neddy Bate Valued Senior Member

    Messages:
    2,548
    If you are telling the embankment frame that they are not allowed to use the Lorentz transforms to determine that the head and feet impact at the same time in their own frame, then I disagree.


    My explanation is consistent with his. I don't know what he means by "Falls straight down", because the body will be tilted on an angle as it falls, and the direction of the fall will not be vertical, according to the embankment frame. But his explanation for why the head will not absorb the whole impact is consistent with my explanation.
     
  22. Tach Banned Banned

    Messages:
    5,265
    A rare admission of error from you, must mark the calendar.



    So?


    How is this different from what Markus is saying?
     
  23. Tach Banned Banned

    Messages:
    5,265
    That's not what I am telling you, what I am telling you is that you are trying to use a sleigh of hand to re-transform the problem in the train frame. You are an observer in the embankment frame, you must judge the experiment based on what you observe in the embankment frame, consider the fact that you can't see what happens in the train frame.





    No, it isn't. "Straight down" for Markus means parallel with the ground, un-rotated. The Thomas precession cancels out the effects of RoS. You are considering RoS only (and trying to re-transform the problem back into the frame of the car through an inverse Lorentz transform, where the ends strike the floor simultaneously). Nowhere in your posts does the Thomas precession show up.
     

Share This Page